Difference between revisions of "2021 Fall AMC 12A Problems/Problem 15"
(→Solution) |
(→Solution) |
||
Line 14: | Line 14: | ||
− | Our answer is <math>B+D=256-48=\boxed{(\textbf{D}) \: | + | Our answer is <math>B+D=256-48=\boxed{(\textbf{D}) \: 208}.</math> |
~kingofpineapplz | ~kingofpineapplz |
Latest revision as of 16:06, 1 November 2024
Problem
Recall that the conjugate of the complex number , where and are real numbers and , is the complex number . For any complex number , let . The polynomial has four complex roots: , , , and . Let be the polynomial whose roots are , , , and , where the coefficients and are complex numbers. What is
Solution
By Vieta's formulas, , and
Since Since
Our answer is
~kingofpineapplz
See Also
2021 Fall AMC 12A (Problems • Answer Key • Resources) | |
Preceded by Problem 14 |
Followed by Problem 16 |
1 • 2 • 3 • 4 • 5 • 6 • 7 • 8 • 9 • 10 • 11 • 12 • 13 • 14 • 15 • 16 • 17 • 18 • 19 • 20 • 21 • 22 • 23 • 24 • 25 | |
All AMC 12 Problems and Solutions |
The problems on this page are copyrighted by the Mathematical Association of America's American Mathematics Competitions.